Modelo de Potts 2D: mudanças entre as edições

De Física Computacional
Ir para navegação Ir para pesquisar
Linha 14: Linha 14:
nesse caso os spins <math>s_i</math> e <math>s_j</math> têm apenas dois valores possíveis e
nesse caso os spins <math>s_i</math> e <math>s_j</math> têm apenas dois valores possíveis e


<math> 2\delta(s_i,s_j) - 1 = \begin{cases}
<math> 2\delta(s_i,s_j) - 1 = \begin{cases}
  1, \quad \text{se } s_i = s_j \\
  1, \quad \text{se } s_i = s_j \\
  -1, \quad \text{se } s_i \neq s_j
  -1, \quad \text{se } s_i \neq s_j
\end{cases}</math>
\end{cases}</math>
 
logo considerando como valores possíveis para os spins <math>\{s_i,s_j\}</math> como <math>-1</math> ou <math>1</math> encontramos
 
<math>H_I = H_p + \sum_{(i,j)}\frac{J}{2} = -\frac{J}{2}\sum_{(i,j)} s_i s_j </math>

Edição das 17h42min de 9 de maio de 2021

Modelo de Potts

O "Modelo de Potts de Q-estados" trata de um sistema de rede com N spins interagentes , onde um spin pode assumir um valor inteiro e positivo . Cada spin do sistema está limitado a interagir com outros spins em sua vizinhança e a energia da interação entre dois spins e é dada pelo potencial

onde é a função delta de Kronecker e é a constante de interação entre os spins. Dessa maneira, a interação entre dois spins vizinhos contabiliza um valor de energia ao sistema apenas se . A hamiltoniana do sistema é dada pela soma entre todas as interações entre spins vizinhos:

Este modelo é tido como uma generalização natural do Modelo de Ising e para ambos modelos são equivalentes a menos de uma constante:

nesse caso os spins e têm apenas dois valores possíveis e